If $(a_n)toinfty$, then $(1/a_n)to0$












2












$begingroup$



Prove: If $(a_n)toinfty$, then $(1/a_n)to0$




As $a_ntoinfty$, for every $C>0$ there exists an $N$ in the natural numbers such that we have



$a_n>C$ whenever $n>N$



$implies|a_n|>C$ whenever $n>N$



$implies|1/a_n| < 1/C$ whenever $n>N$



If follows that if we choose $e>0$ there exists an $N_1$ such that we have



$|1/a_n| < e$ whenever $n>N_1$



How is this proof, is it acceptable do you think?










share|cite|improve this question











$endgroup$








  • 1




    $begingroup$
    Almost. For $varepsilon$ choose $C:=1/varepsilon$
    $endgroup$
    – Berci
    Jan 10 at 17:43










  • $begingroup$
    Thanks I don't quite understand are you saying that for the last bit I should say that if we choose e=1/C the definition will be satisfied?
    $endgroup$
    – James Doherty
    Jan 10 at 17:50










  • $begingroup$
    Yes, I miss that part from the proof.. But not exactly.. First, $varepsilon$ is given, and we're looking for an $N_1$, which will be chosen for $C=1/varepsilon$ by the hypothesis.
    $endgroup$
    – Berci
    Jan 10 at 17:51












  • $begingroup$
    @JamesDoherty You need to show that $forallepsilon>0,exists n_0inBbb N:|1/a_n|<epsilonforall nge n_0$. To prove this, take $C=1/epsilon>0$. Then $exists n_1inBbb N:a_n>Cforall nge n_1implies 1/a_n=|1/a_n|<1/C=epsilonforall nge n_0=n_1$
    $endgroup$
    – Shubham Johri
    Jan 10 at 17:56


















2












$begingroup$



Prove: If $(a_n)toinfty$, then $(1/a_n)to0$




As $a_ntoinfty$, for every $C>0$ there exists an $N$ in the natural numbers such that we have



$a_n>C$ whenever $n>N$



$implies|a_n|>C$ whenever $n>N$



$implies|1/a_n| < 1/C$ whenever $n>N$



If follows that if we choose $e>0$ there exists an $N_1$ such that we have



$|1/a_n| < e$ whenever $n>N_1$



How is this proof, is it acceptable do you think?










share|cite|improve this question











$endgroup$








  • 1




    $begingroup$
    Almost. For $varepsilon$ choose $C:=1/varepsilon$
    $endgroup$
    – Berci
    Jan 10 at 17:43










  • $begingroup$
    Thanks I don't quite understand are you saying that for the last bit I should say that if we choose e=1/C the definition will be satisfied?
    $endgroup$
    – James Doherty
    Jan 10 at 17:50










  • $begingroup$
    Yes, I miss that part from the proof.. But not exactly.. First, $varepsilon$ is given, and we're looking for an $N_1$, which will be chosen for $C=1/varepsilon$ by the hypothesis.
    $endgroup$
    – Berci
    Jan 10 at 17:51












  • $begingroup$
    @JamesDoherty You need to show that $forallepsilon>0,exists n_0inBbb N:|1/a_n|<epsilonforall nge n_0$. To prove this, take $C=1/epsilon>0$. Then $exists n_1inBbb N:a_n>Cforall nge n_1implies 1/a_n=|1/a_n|<1/C=epsilonforall nge n_0=n_1$
    $endgroup$
    – Shubham Johri
    Jan 10 at 17:56
















2












2








2





$begingroup$



Prove: If $(a_n)toinfty$, then $(1/a_n)to0$




As $a_ntoinfty$, for every $C>0$ there exists an $N$ in the natural numbers such that we have



$a_n>C$ whenever $n>N$



$implies|a_n|>C$ whenever $n>N$



$implies|1/a_n| < 1/C$ whenever $n>N$



If follows that if we choose $e>0$ there exists an $N_1$ such that we have



$|1/a_n| < e$ whenever $n>N_1$



How is this proof, is it acceptable do you think?










share|cite|improve this question











$endgroup$





Prove: If $(a_n)toinfty$, then $(1/a_n)to0$




As $a_ntoinfty$, for every $C>0$ there exists an $N$ in the natural numbers such that we have



$a_n>C$ whenever $n>N$



$implies|a_n|>C$ whenever $n>N$



$implies|1/a_n| < 1/C$ whenever $n>N$



If follows that if we choose $e>0$ there exists an $N_1$ such that we have



$|1/a_n| < e$ whenever $n>N_1$



How is this proof, is it acceptable do you think?







proof-verification






share|cite|improve this question















share|cite|improve this question













share|cite|improve this question




share|cite|improve this question








edited Jan 10 at 17:49









Shubham Johri

5,102717




5,102717










asked Jan 10 at 17:28









James DohertyJames Doherty

527




527








  • 1




    $begingroup$
    Almost. For $varepsilon$ choose $C:=1/varepsilon$
    $endgroup$
    – Berci
    Jan 10 at 17:43










  • $begingroup$
    Thanks I don't quite understand are you saying that for the last bit I should say that if we choose e=1/C the definition will be satisfied?
    $endgroup$
    – James Doherty
    Jan 10 at 17:50










  • $begingroup$
    Yes, I miss that part from the proof.. But not exactly.. First, $varepsilon$ is given, and we're looking for an $N_1$, which will be chosen for $C=1/varepsilon$ by the hypothesis.
    $endgroup$
    – Berci
    Jan 10 at 17:51












  • $begingroup$
    @JamesDoherty You need to show that $forallepsilon>0,exists n_0inBbb N:|1/a_n|<epsilonforall nge n_0$. To prove this, take $C=1/epsilon>0$. Then $exists n_1inBbb N:a_n>Cforall nge n_1implies 1/a_n=|1/a_n|<1/C=epsilonforall nge n_0=n_1$
    $endgroup$
    – Shubham Johri
    Jan 10 at 17:56
















  • 1




    $begingroup$
    Almost. For $varepsilon$ choose $C:=1/varepsilon$
    $endgroup$
    – Berci
    Jan 10 at 17:43










  • $begingroup$
    Thanks I don't quite understand are you saying that for the last bit I should say that if we choose e=1/C the definition will be satisfied?
    $endgroup$
    – James Doherty
    Jan 10 at 17:50










  • $begingroup$
    Yes, I miss that part from the proof.. But not exactly.. First, $varepsilon$ is given, and we're looking for an $N_1$, which will be chosen for $C=1/varepsilon$ by the hypothesis.
    $endgroup$
    – Berci
    Jan 10 at 17:51












  • $begingroup$
    @JamesDoherty You need to show that $forallepsilon>0,exists n_0inBbb N:|1/a_n|<epsilonforall nge n_0$. To prove this, take $C=1/epsilon>0$. Then $exists n_1inBbb N:a_n>Cforall nge n_1implies 1/a_n=|1/a_n|<1/C=epsilonforall nge n_0=n_1$
    $endgroup$
    – Shubham Johri
    Jan 10 at 17:56










1




1




$begingroup$
Almost. For $varepsilon$ choose $C:=1/varepsilon$
$endgroup$
– Berci
Jan 10 at 17:43




$begingroup$
Almost. For $varepsilon$ choose $C:=1/varepsilon$
$endgroup$
– Berci
Jan 10 at 17:43












$begingroup$
Thanks I don't quite understand are you saying that for the last bit I should say that if we choose e=1/C the definition will be satisfied?
$endgroup$
– James Doherty
Jan 10 at 17:50




$begingroup$
Thanks I don't quite understand are you saying that for the last bit I should say that if we choose e=1/C the definition will be satisfied?
$endgroup$
– James Doherty
Jan 10 at 17:50












$begingroup$
Yes, I miss that part from the proof.. But not exactly.. First, $varepsilon$ is given, and we're looking for an $N_1$, which will be chosen for $C=1/varepsilon$ by the hypothesis.
$endgroup$
– Berci
Jan 10 at 17:51






$begingroup$
Yes, I miss that part from the proof.. But not exactly.. First, $varepsilon$ is given, and we're looking for an $N_1$, which will be chosen for $C=1/varepsilon$ by the hypothesis.
$endgroup$
– Berci
Jan 10 at 17:51














$begingroup$
@JamesDoherty You need to show that $forallepsilon>0,exists n_0inBbb N:|1/a_n|<epsilonforall nge n_0$. To prove this, take $C=1/epsilon>0$. Then $exists n_1inBbb N:a_n>Cforall nge n_1implies 1/a_n=|1/a_n|<1/C=epsilonforall nge n_0=n_1$
$endgroup$
– Shubham Johri
Jan 10 at 17:56






$begingroup$
@JamesDoherty You need to show that $forallepsilon>0,exists n_0inBbb N:|1/a_n|<epsilonforall nge n_0$. To prove this, take $C=1/epsilon>0$. Then $exists n_1inBbb N:a_n>Cforall nge n_1implies 1/a_n=|1/a_n|<1/C=epsilonforall nge n_0=n_1$
$endgroup$
– Shubham Johri
Jan 10 at 17:56












0






active

oldest

votes











Your Answer





StackExchange.ifUsing("editor", function () {
return StackExchange.using("mathjaxEditing", function () {
StackExchange.MarkdownEditor.creationCallbacks.add(function (editor, postfix) {
StackExchange.mathjaxEditing.prepareWmdForMathJax(editor, postfix, [["$", "$"], ["\\(","\\)"]]);
});
});
}, "mathjax-editing");

StackExchange.ready(function() {
var channelOptions = {
tags: "".split(" "),
id: "69"
};
initTagRenderer("".split(" "), "".split(" "), channelOptions);

StackExchange.using("externalEditor", function() {
// Have to fire editor after snippets, if snippets enabled
if (StackExchange.settings.snippets.snippetsEnabled) {
StackExchange.using("snippets", function() {
createEditor();
});
}
else {
createEditor();
}
});

function createEditor() {
StackExchange.prepareEditor({
heartbeatType: 'answer',
autoActivateHeartbeat: false,
convertImagesToLinks: true,
noModals: true,
showLowRepImageUploadWarning: true,
reputationToPostImages: 10,
bindNavPrevention: true,
postfix: "",
imageUploader: {
brandingHtml: "Powered by u003ca class="icon-imgur-white" href="https://imgur.com/"u003eu003c/au003e",
contentPolicyHtml: "User contributions licensed under u003ca href="https://creativecommons.org/licenses/by-sa/3.0/"u003ecc by-sa 3.0 with attribution requiredu003c/au003e u003ca href="https://stackoverflow.com/legal/content-policy"u003e(content policy)u003c/au003e",
allowUrls: true
},
noCode: true, onDemand: true,
discardSelector: ".discard-answer"
,immediatelyShowMarkdownHelp:true
});


}
});














draft saved

draft discarded


















StackExchange.ready(
function () {
StackExchange.openid.initPostLogin('.new-post-login', 'https%3a%2f%2fmath.stackexchange.com%2fquestions%2f3068945%2fif-a-n-to-infty-then-1-a-n-to0%23new-answer', 'question_page');
}
);

Post as a guest















Required, but never shown

























0






active

oldest

votes








0






active

oldest

votes









active

oldest

votes






active

oldest

votes
















draft saved

draft discarded




















































Thanks for contributing an answer to Mathematics Stack Exchange!


  • Please be sure to answer the question. Provide details and share your research!

But avoid



  • Asking for help, clarification, or responding to other answers.

  • Making statements based on opinion; back them up with references or personal experience.


Use MathJax to format equations. MathJax reference.


To learn more, see our tips on writing great answers.




draft saved


draft discarded














StackExchange.ready(
function () {
StackExchange.openid.initPostLogin('.new-post-login', 'https%3a%2f%2fmath.stackexchange.com%2fquestions%2f3068945%2fif-a-n-to-infty-then-1-a-n-to0%23new-answer', 'question_page');
}
);

Post as a guest















Required, but never shown





















































Required, but never shown














Required, but never shown












Required, but never shown







Required, but never shown

































Required, but never shown














Required, but never shown












Required, but never shown







Required, but never shown







Popular posts from this blog

Can a sorcerer learn a 5th-level spell early by creating spell slots using the Font of Magic feature?

Does disintegrating a polymorphed enemy still kill it after the 2018 errata?

A Topological Invariant for $pi_3(U(n))$